complicated answer choice
My pre-phrase for this question was "the economic downturn did not cause banks to loan less." Whi...
Abigail-Okereke on February 23, 2023
  • December 1999 LSAT
  • SEC2
  • Q15
1
Reply
A vs B
I dont see the difference between A and B; to me, they both have the same flaw. Why is B a better...
Abigail-Okereke on January 20, 2023
  • December 1999 LSAT
  • SEC2
  • Q6
1
Reply
B vs C
Why is C a better answer than B. I also didn't realize that this was a paradox question.
Abigail-Okereke on January 20, 2023
  • December 1999 LSAT
  • SEC2
  • Q9
1
Reply
flaw
What is this here and why is D the answer? I was between answer choices A and E.
Abigail-Okereke on January 20, 2023
  • December 1999 LSAT
  • SEC2
  • Q13
1
Reply
Why is the second diagram (sentence) not "not b...
Unless introduces a necessary condition and the sentence states "But there will not be a good sho...
Sammy on August 26, 2022
  • December 1999 LSAT
  • SEC2
  • Q18
2
Replies
Help
To me, (A), (B) and (E) all seem to be supporting for the prediction in the stimulus. Please, exp...
Batman on February 6, 2022
  • December 1999 LSAT
  • SEC2
  • Q21
3
Replies
Why is the answer A?
I diagrammed this question using the transitive property: Inspired performance -> Good Show -> S...
JennaArthur on September 13, 2021
  • December 1999 LSAT
  • SEC2
  • Q18
2
Replies
Why E ?
Why is E a better answer than D ?
Maxx on January 30, 2021
  • December 1999 LSAT
  • SEC2
  • Q2
1
Reply
Why not C ?
Why not C instead of D ?
Maxx on January 30, 2021
  • December 1999 LSAT
  • SEC2
  • Q25
1
Reply
Why not C ?
Why not C instead of D ?
Maxx on January 30, 2021
  • December 1999 LSAT
  • SEC2
  • Q25
1
Reply
Why is D correct ?
Why D ?
Maxx on January 30, 2021
  • December 1999 LSAT
  • SEC2
  • Q23
1
Reply
If versus To Be
When I began diagramming for this question, I was really confused as to how to diagram for "and t...
miriamelise on January 3, 2021
  • December 1999 LSAT
  • SEC2
  • Q18
1
Reply
Explain?
Can someone please explain this one for me?
Kweb1016 on December 21, 2020
  • December 1999 LSAT
  • SEC2
  • Q24
1
Reply
“Unless” S&N
Hello! I got to the correct conclusion and answer in the wrong way. I diagrammed GoodShow—>Sophsi...
aliH on November 7, 2020
  • December 1999 LSAT
  • SEC2
  • Q18
1
Reply
Questions
Please could you explain the wording in answer choice D - I didn't pick this because I didn't und...
Anna20 on August 11, 2020
  • December 1999 LSAT
  • SEC2
  • Q7
1
Reply
Why is B incorrect?
Why is B incorrect?
Shiyi-Zhang on July 19, 2020
  • December 1999 LSAT
  • SEC2
  • Q15
2
Replies
Could someone please explain this?
Could someone please explain this? Thanks
jingjingxiao11111@gmail.com on June 26, 2020
  • December 1999 LSAT
  • SEC2
  • Q26
1
Reply
Why not B?
When I diagrammed the question I got IPC => GS => SL => UMR. For B I have not UMR => not GS ...
Joyce on March 20, 2020
  • December 1999 LSAT
  • SEC2
  • Q18
3
Replies
Could you explain the flaw here and the correct...
I actually love flaw questions but it seems they don t love me back. Before I get to the questio...
Henleys on March 19, 2020
  • December 1999 LSAT
  • SEC2
  • Q17
3
Replies
IPC without GS?
Isn't it possible to have inspired performances without it being a good show? Does the fact of re...
whlocke on January 6, 2020
  • December 1999 LSAT
  • SEC2
  • Q18
1
Reply